How to solve this differential equation with singularity?












2












$begingroup$


I want to find the solution of the following differential equation.



$$
frac{mathrm{d}^{2}x}{mathrm{d}t^{2}} =
−frac{1}{ax + mathrm{d}x/mathrm{d}t}
$$



where a is a positive constant and x is a position of a free object that is not bounded or tied to a fixed object (e.g., a wall).



I think that the solution may behave like an oscillator, but it seem to be slightly different from that of the oscillator. However, due to the terms in the denominator, it has the singularity. How to solve this type of equation?



** As I studied differential equations for a while 20 years ago, I do not know how to solve this problem. Since the solution of the above differential equation is an important issue for me, I would appreciate it if you help.










share|cite|improve this question











$endgroup$

















    2












    $begingroup$


    I want to find the solution of the following differential equation.



    $$
    frac{mathrm{d}^{2}x}{mathrm{d}t^{2}} =
    −frac{1}{ax + mathrm{d}x/mathrm{d}t}
    $$



    where a is a positive constant and x is a position of a free object that is not bounded or tied to a fixed object (e.g., a wall).



    I think that the solution may behave like an oscillator, but it seem to be slightly different from that of the oscillator. However, due to the terms in the denominator, it has the singularity. How to solve this type of equation?



    ** As I studied differential equations for a while 20 years ago, I do not know how to solve this problem. Since the solution of the above differential equation is an important issue for me, I would appreciate it if you help.










    share|cite|improve this question











    $endgroup$















      2












      2








      2


      3



      $begingroup$


      I want to find the solution of the following differential equation.



      $$
      frac{mathrm{d}^{2}x}{mathrm{d}t^{2}} =
      −frac{1}{ax + mathrm{d}x/mathrm{d}t}
      $$



      where a is a positive constant and x is a position of a free object that is not bounded or tied to a fixed object (e.g., a wall).



      I think that the solution may behave like an oscillator, but it seem to be slightly different from that of the oscillator. However, due to the terms in the denominator, it has the singularity. How to solve this type of equation?



      ** As I studied differential equations for a while 20 years ago, I do not know how to solve this problem. Since the solution of the above differential equation is an important issue for me, I would appreciate it if you help.










      share|cite|improve this question











      $endgroup$




      I want to find the solution of the following differential equation.



      $$
      frac{mathrm{d}^{2}x}{mathrm{d}t^{2}} =
      −frac{1}{ax + mathrm{d}x/mathrm{d}t}
      $$



      where a is a positive constant and x is a position of a free object that is not bounded or tied to a fixed object (e.g., a wall).



      I think that the solution may behave like an oscillator, but it seem to be slightly different from that of the oscillator. However, due to the terms in the denominator, it has the singularity. How to solve this type of equation?



      ** As I studied differential equations for a while 20 years ago, I do not know how to solve this problem. Since the solution of the above differential equation is an important issue for me, I would appreciate it if you help.







      ordinary-differential-equations wave-equation






      share|cite|improve this question















      share|cite|improve this question













      share|cite|improve this question




      share|cite|improve this question








      edited Jan 6 at 20:16









      Felix Marin

      67.3k7107141




      67.3k7107141










      asked Jan 6 at 12:32









      SOQEHSOQEH

      134




      134






















          1 Answer
          1






          active

          oldest

          votes


















          3












          $begingroup$

          $$frac{d^2x}{dt^2}=−frac{1}{a x + frac{dx}{dt}} tag 1$$
          The usual change of function to reduce the order of this autonomous ODE is



          $frac{dx}{dt}=y(x) quad;quad frac{d^2x}{dt^2}=frac{dy}{dx}frac{dx}{dt}=yfrac{dy}{dx}$



          $$yfrac{dy}{dx}=−frac{1}{a x + y}$$



          Consider the inverse function $x(y)$
          $$frac{dx}{dy}=−(a x + y)y$$
          This is a first order linear ODE with respect to the function $x(y)$. The solution is :
          $$x(y)=-frac{y}{a}+sqrt{frac{pi}{2a^3}}e^{-ay^2/2}text{erfi}
          left(sqrt{frac{a}{2}}:yright)+c_1 e^{-ay^2/2}$$



          Unfortunately there is no closed form for the inverse function $y(x)$ in general (for arbitrary values of $a$ and $c_1$).



          A fortiori no closed form for $t=intfrac{dx}{y(x)}$.



          Of course, the solutions of equation $(1)$ exist but in general they cannot be expressed with a finite number of standard functions.



          As a consequence, in practice solve the ODE $(1)$ with numerical calculus. It should be a waste of time to try to solve it analytically in general.



          Nevertheless, it might be possible to solve the problem analytically is some particular cases, depending on the initial conditions of the ODE and/or with particular values of the parameter $a$.






          share|cite|improve this answer











          $endgroup$









          • 1




            $begingroup$
            should be $y^{-1}dx/dy=-(ax+y)$
            $endgroup$
            – user254433
            Jan 6 at 20:22










          • $begingroup$
            You are right. Thank you for your remark. I corrected my calculus. Finally the conclusion remains the same.
            $endgroup$
            – JJacquelin
            Jan 6 at 21:41










          • $begingroup$
            @JJacquelin I am much obliged to you for your help.
            $endgroup$
            – SOQEH
            Jan 7 at 0:17










          • $begingroup$
            I don't think $x=-y/a$ is a solution
            $endgroup$
            – Dylan
            Jan 7 at 9:16






          • 1




            $begingroup$
            Hi Jean ! You probably noticed that, assuming $a>0$ and small values of $y$, we have $x(y)=-frac{y^3}{3}+frac{a y^5}{15}+Oleft(y^7right)$ and then we could (may be !) get ... something. Cheers.
            $endgroup$
            – Claude Leibovici
            Jan 7 at 11:15











          Your Answer





          StackExchange.ifUsing("editor", function () {
          return StackExchange.using("mathjaxEditing", function () {
          StackExchange.MarkdownEditor.creationCallbacks.add(function (editor, postfix) {
          StackExchange.mathjaxEditing.prepareWmdForMathJax(editor, postfix, [["$", "$"], ["\\(","\\)"]]);
          });
          });
          }, "mathjax-editing");

          StackExchange.ready(function() {
          var channelOptions = {
          tags: "".split(" "),
          id: "69"
          };
          initTagRenderer("".split(" "), "".split(" "), channelOptions);

          StackExchange.using("externalEditor", function() {
          // Have to fire editor after snippets, if snippets enabled
          if (StackExchange.settings.snippets.snippetsEnabled) {
          StackExchange.using("snippets", function() {
          createEditor();
          });
          }
          else {
          createEditor();
          }
          });

          function createEditor() {
          StackExchange.prepareEditor({
          heartbeatType: 'answer',
          autoActivateHeartbeat: false,
          convertImagesToLinks: true,
          noModals: true,
          showLowRepImageUploadWarning: true,
          reputationToPostImages: 10,
          bindNavPrevention: true,
          postfix: "",
          imageUploader: {
          brandingHtml: "Powered by u003ca class="icon-imgur-white" href="https://imgur.com/"u003eu003c/au003e",
          contentPolicyHtml: "User contributions licensed under u003ca href="https://creativecommons.org/licenses/by-sa/3.0/"u003ecc by-sa 3.0 with attribution requiredu003c/au003e u003ca href="https://stackoverflow.com/legal/content-policy"u003e(content policy)u003c/au003e",
          allowUrls: true
          },
          noCode: true, onDemand: true,
          discardSelector: ".discard-answer"
          ,immediatelyShowMarkdownHelp:true
          });


          }
          });














          draft saved

          draft discarded


















          StackExchange.ready(
          function () {
          StackExchange.openid.initPostLogin('.new-post-login', 'https%3a%2f%2fmath.stackexchange.com%2fquestions%2f3063804%2fhow-to-solve-this-differential-equation-with-singularity%23new-answer', 'question_page');
          }
          );

          Post as a guest















          Required, but never shown

























          1 Answer
          1






          active

          oldest

          votes








          1 Answer
          1






          active

          oldest

          votes









          active

          oldest

          votes






          active

          oldest

          votes









          3












          $begingroup$

          $$frac{d^2x}{dt^2}=−frac{1}{a x + frac{dx}{dt}} tag 1$$
          The usual change of function to reduce the order of this autonomous ODE is



          $frac{dx}{dt}=y(x) quad;quad frac{d^2x}{dt^2}=frac{dy}{dx}frac{dx}{dt}=yfrac{dy}{dx}$



          $$yfrac{dy}{dx}=−frac{1}{a x + y}$$



          Consider the inverse function $x(y)$
          $$frac{dx}{dy}=−(a x + y)y$$
          This is a first order linear ODE with respect to the function $x(y)$. The solution is :
          $$x(y)=-frac{y}{a}+sqrt{frac{pi}{2a^3}}e^{-ay^2/2}text{erfi}
          left(sqrt{frac{a}{2}}:yright)+c_1 e^{-ay^2/2}$$



          Unfortunately there is no closed form for the inverse function $y(x)$ in general (for arbitrary values of $a$ and $c_1$).



          A fortiori no closed form for $t=intfrac{dx}{y(x)}$.



          Of course, the solutions of equation $(1)$ exist but in general they cannot be expressed with a finite number of standard functions.



          As a consequence, in practice solve the ODE $(1)$ with numerical calculus. It should be a waste of time to try to solve it analytically in general.



          Nevertheless, it might be possible to solve the problem analytically is some particular cases, depending on the initial conditions of the ODE and/or with particular values of the parameter $a$.






          share|cite|improve this answer











          $endgroup$









          • 1




            $begingroup$
            should be $y^{-1}dx/dy=-(ax+y)$
            $endgroup$
            – user254433
            Jan 6 at 20:22










          • $begingroup$
            You are right. Thank you for your remark. I corrected my calculus. Finally the conclusion remains the same.
            $endgroup$
            – JJacquelin
            Jan 6 at 21:41










          • $begingroup$
            @JJacquelin I am much obliged to you for your help.
            $endgroup$
            – SOQEH
            Jan 7 at 0:17










          • $begingroup$
            I don't think $x=-y/a$ is a solution
            $endgroup$
            – Dylan
            Jan 7 at 9:16






          • 1




            $begingroup$
            Hi Jean ! You probably noticed that, assuming $a>0$ and small values of $y$, we have $x(y)=-frac{y^3}{3}+frac{a y^5}{15}+Oleft(y^7right)$ and then we could (may be !) get ... something. Cheers.
            $endgroup$
            – Claude Leibovici
            Jan 7 at 11:15
















          3












          $begingroup$

          $$frac{d^2x}{dt^2}=−frac{1}{a x + frac{dx}{dt}} tag 1$$
          The usual change of function to reduce the order of this autonomous ODE is



          $frac{dx}{dt}=y(x) quad;quad frac{d^2x}{dt^2}=frac{dy}{dx}frac{dx}{dt}=yfrac{dy}{dx}$



          $$yfrac{dy}{dx}=−frac{1}{a x + y}$$



          Consider the inverse function $x(y)$
          $$frac{dx}{dy}=−(a x + y)y$$
          This is a first order linear ODE with respect to the function $x(y)$. The solution is :
          $$x(y)=-frac{y}{a}+sqrt{frac{pi}{2a^3}}e^{-ay^2/2}text{erfi}
          left(sqrt{frac{a}{2}}:yright)+c_1 e^{-ay^2/2}$$



          Unfortunately there is no closed form for the inverse function $y(x)$ in general (for arbitrary values of $a$ and $c_1$).



          A fortiori no closed form for $t=intfrac{dx}{y(x)}$.



          Of course, the solutions of equation $(1)$ exist but in general they cannot be expressed with a finite number of standard functions.



          As a consequence, in practice solve the ODE $(1)$ with numerical calculus. It should be a waste of time to try to solve it analytically in general.



          Nevertheless, it might be possible to solve the problem analytically is some particular cases, depending on the initial conditions of the ODE and/or with particular values of the parameter $a$.






          share|cite|improve this answer











          $endgroup$









          • 1




            $begingroup$
            should be $y^{-1}dx/dy=-(ax+y)$
            $endgroup$
            – user254433
            Jan 6 at 20:22










          • $begingroup$
            You are right. Thank you for your remark. I corrected my calculus. Finally the conclusion remains the same.
            $endgroup$
            – JJacquelin
            Jan 6 at 21:41










          • $begingroup$
            @JJacquelin I am much obliged to you for your help.
            $endgroup$
            – SOQEH
            Jan 7 at 0:17










          • $begingroup$
            I don't think $x=-y/a$ is a solution
            $endgroup$
            – Dylan
            Jan 7 at 9:16






          • 1




            $begingroup$
            Hi Jean ! You probably noticed that, assuming $a>0$ and small values of $y$, we have $x(y)=-frac{y^3}{3}+frac{a y^5}{15}+Oleft(y^7right)$ and then we could (may be !) get ... something. Cheers.
            $endgroup$
            – Claude Leibovici
            Jan 7 at 11:15














          3












          3








          3





          $begingroup$

          $$frac{d^2x}{dt^2}=−frac{1}{a x + frac{dx}{dt}} tag 1$$
          The usual change of function to reduce the order of this autonomous ODE is



          $frac{dx}{dt}=y(x) quad;quad frac{d^2x}{dt^2}=frac{dy}{dx}frac{dx}{dt}=yfrac{dy}{dx}$



          $$yfrac{dy}{dx}=−frac{1}{a x + y}$$



          Consider the inverse function $x(y)$
          $$frac{dx}{dy}=−(a x + y)y$$
          This is a first order linear ODE with respect to the function $x(y)$. The solution is :
          $$x(y)=-frac{y}{a}+sqrt{frac{pi}{2a^3}}e^{-ay^2/2}text{erfi}
          left(sqrt{frac{a}{2}}:yright)+c_1 e^{-ay^2/2}$$



          Unfortunately there is no closed form for the inverse function $y(x)$ in general (for arbitrary values of $a$ and $c_1$).



          A fortiori no closed form for $t=intfrac{dx}{y(x)}$.



          Of course, the solutions of equation $(1)$ exist but in general they cannot be expressed with a finite number of standard functions.



          As a consequence, in practice solve the ODE $(1)$ with numerical calculus. It should be a waste of time to try to solve it analytically in general.



          Nevertheless, it might be possible to solve the problem analytically is some particular cases, depending on the initial conditions of the ODE and/or with particular values of the parameter $a$.






          share|cite|improve this answer











          $endgroup$



          $$frac{d^2x}{dt^2}=−frac{1}{a x + frac{dx}{dt}} tag 1$$
          The usual change of function to reduce the order of this autonomous ODE is



          $frac{dx}{dt}=y(x) quad;quad frac{d^2x}{dt^2}=frac{dy}{dx}frac{dx}{dt}=yfrac{dy}{dx}$



          $$yfrac{dy}{dx}=−frac{1}{a x + y}$$



          Consider the inverse function $x(y)$
          $$frac{dx}{dy}=−(a x + y)y$$
          This is a first order linear ODE with respect to the function $x(y)$. The solution is :
          $$x(y)=-frac{y}{a}+sqrt{frac{pi}{2a^3}}e^{-ay^2/2}text{erfi}
          left(sqrt{frac{a}{2}}:yright)+c_1 e^{-ay^2/2}$$



          Unfortunately there is no closed form for the inverse function $y(x)$ in general (for arbitrary values of $a$ and $c_1$).



          A fortiori no closed form for $t=intfrac{dx}{y(x)}$.



          Of course, the solutions of equation $(1)$ exist but in general they cannot be expressed with a finite number of standard functions.



          As a consequence, in practice solve the ODE $(1)$ with numerical calculus. It should be a waste of time to try to solve it analytically in general.



          Nevertheless, it might be possible to solve the problem analytically is some particular cases, depending on the initial conditions of the ODE and/or with particular values of the parameter $a$.







          share|cite|improve this answer














          share|cite|improve this answer



          share|cite|improve this answer








          edited Jan 6 at 21:39

























          answered Jan 6 at 18:31









          JJacquelinJJacquelin

          43k21751




          43k21751








          • 1




            $begingroup$
            should be $y^{-1}dx/dy=-(ax+y)$
            $endgroup$
            – user254433
            Jan 6 at 20:22










          • $begingroup$
            You are right. Thank you for your remark. I corrected my calculus. Finally the conclusion remains the same.
            $endgroup$
            – JJacquelin
            Jan 6 at 21:41










          • $begingroup$
            @JJacquelin I am much obliged to you for your help.
            $endgroup$
            – SOQEH
            Jan 7 at 0:17










          • $begingroup$
            I don't think $x=-y/a$ is a solution
            $endgroup$
            – Dylan
            Jan 7 at 9:16






          • 1




            $begingroup$
            Hi Jean ! You probably noticed that, assuming $a>0$ and small values of $y$, we have $x(y)=-frac{y^3}{3}+frac{a y^5}{15}+Oleft(y^7right)$ and then we could (may be !) get ... something. Cheers.
            $endgroup$
            – Claude Leibovici
            Jan 7 at 11:15














          • 1




            $begingroup$
            should be $y^{-1}dx/dy=-(ax+y)$
            $endgroup$
            – user254433
            Jan 6 at 20:22










          • $begingroup$
            You are right. Thank you for your remark. I corrected my calculus. Finally the conclusion remains the same.
            $endgroup$
            – JJacquelin
            Jan 6 at 21:41










          • $begingroup$
            @JJacquelin I am much obliged to you for your help.
            $endgroup$
            – SOQEH
            Jan 7 at 0:17










          • $begingroup$
            I don't think $x=-y/a$ is a solution
            $endgroup$
            – Dylan
            Jan 7 at 9:16






          • 1




            $begingroup$
            Hi Jean ! You probably noticed that, assuming $a>0$ and small values of $y$, we have $x(y)=-frac{y^3}{3}+frac{a y^5}{15}+Oleft(y^7right)$ and then we could (may be !) get ... something. Cheers.
            $endgroup$
            – Claude Leibovici
            Jan 7 at 11:15








          1




          1




          $begingroup$
          should be $y^{-1}dx/dy=-(ax+y)$
          $endgroup$
          – user254433
          Jan 6 at 20:22




          $begingroup$
          should be $y^{-1}dx/dy=-(ax+y)$
          $endgroup$
          – user254433
          Jan 6 at 20:22












          $begingroup$
          You are right. Thank you for your remark. I corrected my calculus. Finally the conclusion remains the same.
          $endgroup$
          – JJacquelin
          Jan 6 at 21:41




          $begingroup$
          You are right. Thank you for your remark. I corrected my calculus. Finally the conclusion remains the same.
          $endgroup$
          – JJacquelin
          Jan 6 at 21:41












          $begingroup$
          @JJacquelin I am much obliged to you for your help.
          $endgroup$
          – SOQEH
          Jan 7 at 0:17




          $begingroup$
          @JJacquelin I am much obliged to you for your help.
          $endgroup$
          – SOQEH
          Jan 7 at 0:17












          $begingroup$
          I don't think $x=-y/a$ is a solution
          $endgroup$
          – Dylan
          Jan 7 at 9:16




          $begingroup$
          I don't think $x=-y/a$ is a solution
          $endgroup$
          – Dylan
          Jan 7 at 9:16




          1




          1




          $begingroup$
          Hi Jean ! You probably noticed that, assuming $a>0$ and small values of $y$, we have $x(y)=-frac{y^3}{3}+frac{a y^5}{15}+Oleft(y^7right)$ and then we could (may be !) get ... something. Cheers.
          $endgroup$
          – Claude Leibovici
          Jan 7 at 11:15




          $begingroup$
          Hi Jean ! You probably noticed that, assuming $a>0$ and small values of $y$, we have $x(y)=-frac{y^3}{3}+frac{a y^5}{15}+Oleft(y^7right)$ and then we could (may be !) get ... something. Cheers.
          $endgroup$
          – Claude Leibovici
          Jan 7 at 11:15


















          draft saved

          draft discarded




















































          Thanks for contributing an answer to Mathematics Stack Exchange!


          • Please be sure to answer the question. Provide details and share your research!

          But avoid



          • Asking for help, clarification, or responding to other answers.

          • Making statements based on opinion; back them up with references or personal experience.


          Use MathJax to format equations. MathJax reference.


          To learn more, see our tips on writing great answers.




          draft saved


          draft discarded














          StackExchange.ready(
          function () {
          StackExchange.openid.initPostLogin('.new-post-login', 'https%3a%2f%2fmath.stackexchange.com%2fquestions%2f3063804%2fhow-to-solve-this-differential-equation-with-singularity%23new-answer', 'question_page');
          }
          );

          Post as a guest















          Required, but never shown





















































          Required, but never shown














          Required, but never shown












          Required, but never shown







          Required, but never shown

































          Required, but never shown














          Required, but never shown












          Required, but never shown







          Required, but never shown







          Popular posts from this blog

          An IMO inspired problem

          Management

          Has there ever been an instance of an active nuclear power plant within or near a war zone?